Тёмный

Tree Ordering | LSAT Logic Games 

LSAT Lab
Подписаться 18 тыс.
Просмотров 26 тыс.
50% 1

The rules all come together in Tree Ordering games. Connect the rules to build trees that help you see implicit relationships within the game.
Get the power of personalized learning when you take a free practice digital LSAT.
lsatlab.com/ls...
Get our latest videos when you subscribe to the LSAT Lab RU-vid channel.

Опубликовано:

 

13 сен 2024

Поделиться:

Ссылка:

Скачать:

Готовим ссылку...

Добавить в:

Мой плейлист
Посмотреть позже
Комментарии : 47   
@sharisanders8536
@sharisanders8536 3 года назад
I love the LSAT Lab lessons; the clarity is amazing. It took me SO LONG to understand that I don't have to populate every element / slot, before checking the answers. Sometimes, it's so simple - it's just checking to see if I triggered a rule correctly. Not all 100 steps, just step 1 to step 2. Looking forward to the rest!!
@aarongrubbs5668
@aarongrubbs5668 Год назад
I am continually amazed at how easy these games become if you just know how to set up the rules from the get go. Thank you so much for these videos!
@EconAndExcel
@EconAndExcel 3 года назад
Me before this video: How the hell am I supposed to understand this!? Me after this video: How the hell did I not understand this before? lol
@LSATLab
@LSATLab 3 года назад
That's exactly it. It's kinda like you're in a big dark room at the beginning of LSAT prep, but slowly light bulbs turn on one at a time and everything just becomes clear.
@chukaokeke8504
@chukaokeke8504 Год назад
For me, this is the starting point for understanding LG basics. God bless LSAT Lab!
@dlaky17
@dlaky17 3 года назад
Yours is so far the best sequencing strategies video, and I watched a lot of them. Thanks!!
@nezsa628
@nezsa628 3 года назад
I learn so much from you guys !
@surpriseimblack
@surpriseimblack 10 месяцев назад
You're doing the Lord's work!
@peacescott4123
@peacescott4123 3 года назад
Thank you so much! All of your content is very helpful!!
@jasonlou2013S
@jasonlou2013S 2 года назад
Thank you! Tomorrow I am going to take LSAT!
@brandonsavin13
@brandonsavin13 Год назад
How'd it go??
@jasonlou2013S
@jasonlou2013S Год назад
@@brandonsavin13 I got 145.
@brandonsavin13
@brandonsavin13 Год назад
@@jasonlou2013S That's about where I'm practice testing right now. Did you take it again?
@jasonlou2013S
@jasonlou2013S Год назад
@brandonsavin13 Nope. My family just came to the state so I need to work and help them now. I will take lsat again in the future.
@fit2642
@fit2642 3 года назад
Matt, You're the GOAT!
@andreasmith2258
@andreasmith2258 3 года назад
Wow! I really found this video useful. thank you!
@Mycole
@Mycole 2 года назад
I'm confused with rules between Q2 and Q4. Q2 was explained that tracing relationship between K and G was not possible, since you'd have to shift right, left, right (K >J>F>L>G). However, in Q4, you are assuming the relationship between J and G (that J is released earlier than G) even though you are tracing from different relationships (K>L>G). I hope this makes sense, thanks!
@LSATLab
@LSATLab 2 года назад
Sure, I think the confusion is just the two different tasks. When the path between two letters involves left AND right turns, then the two letters have no relationship in the game. Either one could be ahead or behind of the other. When the path between two letters ONLY involves going left or going right, then they do have a definite ordering relationship. In Q2, we were saying, "Since K and G are connected by a zig-zag path, they have no relationship, so either one could be ahead or behind the other, so it doesn't HAVE to be true that K is before G. (It COULD be true, but it doesn't HAVE to be). In Q4, we are saying, "Since J and G are connected by a zig-zag path, they have no relationship, so either one could be ahead of or behind the other, so it COULD be true that J is earlier than G (it COULD be true, but it doesn't HAVE to be)." Q2 was asking what MUST BE TRUE, and zig-zag buddies don't have any must be true relationship. Either one can be ahead of or behind the other. Q4 was asking what COULD BE TRUE, and zig-zag buddies can be ahead of or behind the other. Either way is possible.
@Mycole
@Mycole 2 года назад
@@LSATLab You're the best. Thank you for detailed response!
@thevanessakay
@thevanessakay Год назад
Wow, this was great to learn! Would you only use this if there are no complex rules such as conditionals?
@benjaminatic9908
@benjaminatic9908 3 года назад
Thank you for starting to upload games! Will you be uploading problems and practices for the other sections as well?
@LSATLab
@LSATLab 3 года назад
Indeed. We'll be producing one LG and one RC video each week for the next 12 weeks.
@benjaminatic9908
@benjaminatic9908 3 года назад
@@LSATLab ok my test is in January, bless y'all!!
@dennisl501
@dennisl501 2 года назад
@@LSATLab It appears that there may not be 12 more LG’s.
@savinajoreva9516
@savinajoreva9516 4 месяца назад
It is visible in the rules that k can not be 5 it has h and j after itself and the films are six.
@carlmarcus8460
@carlmarcus8460 2 года назад
Completely Helpful!
@hornitorrincoperezoso2189
@hornitorrincoperezoso2189 Год назад
When you create the numerical distribution @20:40, that assumes you would also have a rule that each player goes at least once, correct? If all you had was a rule that each element has at least one player, then you could use a 0-0-0-0-5 distribution or anything else that included zeros.
@LSATLab
@LSATLab Год назад
Correct, if they never said "each slot/position has at least one player/element", then you'd have distribution possibilities like 0-0-0-0-7. But in this video, it wasn't assumed that each player/element goes at least once. It was actually specified on screen "each element is assigned to exactly one position", and "each position has at least one element". He was using player and element interchangeably, so that may have gotten confusing.
@kshitijain4829
@kshitijain4829 3 года назад
I have been preparing for 15 days only and have my exam on 30th, will I be able to do good? Someone please help me!
@LSATLab
@LSATLab 3 года назад
Everyone is different. I know someone who scored a 180 on her first practice test. For her, the answer would be yes. I know someone else who spent two years studying for the LSAT. For her, the answer would be no. If you're studying almost every day for the next 15 days, you can probably pull your score up at least 5 points from where it is now.
@kshitijain4829
@kshitijain4829 3 года назад
@@LSATLab I am getting only 40-45 raw score in lsat mock tests
@adarivera5451
@adarivera5451 3 года назад
Good luck on your exam.
@PhenomenalBomb
@PhenomenalBomb 2 года назад
for 15:34 I’m confused… F Is before L so why did you put L in 1? Wouldn’t F being in 2 force K into 1?? Can u plz clarify, thx!!
@therealsherminator
@therealsherminator 2 года назад
Hi, happy to help! At 15:34, we're looking at a Substitution question. The reason we put L in one is because the strategy on Substitution questions relies on creating counterexamples. Notice the rule being substituted is the rule that puts F before L. What we're trying to do at that point in the video is check to see whether answer choice (C) guarantees the original rule. If we can create a counterexample, then the answer choice does not guarantee the rule and we can eliminate the answer choice. So by showing that L could be first, we establish that answer choice (C) does not have the same impact as the original rule.
@PhenomenalBomb
@PhenomenalBomb 2 года назад
@@therealsherminator that makes so much sense! Thank you for taking the time to reply 🙏🏼
@LSATLab
@LSATLab 2 года назад
@@NP-zd6um Why would K have to come before F if F is second? Without the original rule, L could be first. Since L cannot be first with the original rule, answer choice (C) permits a hypothetical the original rules do not.
@LSATLab
@LSATLab 2 года назад
@@NP-zd6um Remember, the rule that F comes before J and L is the rule being substituted. It doesn't exist for this question.
@thevanessakay
@thevanessakay Год назад
Woah, that rule-sub question kicked my butt because I really thought it was C. I still don't understand how it's not C even after listening to the correction a few times.
@thevanessakay
@thevanessakay Год назад
Oh, I see someone else had this question, and it's been answered. Nevermind!
@thevanessakay
@thevanessakay Год назад
This is still mind-boggling. A tutor taught me to test the answer choice out by testing the opposite to challenge it and if it forces that rule by not being able to successfully complete it, then it's correct. But If it works out, then it's wrong. Therefore, when I tested the opposite of answer choice A by putting Fiesta before Kangaroo, I was able to do FKJHLG which worked. So, that is why I eliminated it. Now, I'm just confused.
@LSATLab
@LSATLab Год назад
@@thevanessakay Substitution questions are asking, "which of these NEW rules could be swapped for this OLD rule, such that we'd have the exact same game?" Answers are wrong if either of these things apply: 1. there's a scenario that was LEGAL under the Old rule but is ILLEGAL under the New rule 2. there's a scenario that was ILLEGAL under the Old rule but is LEGAL under the New rule So choice (C) is wrong for reason #2. Consider this scenario L F K J H G This is ILLEGAL under the Old rule (which said that F must be earlier than J and L) but it's LEGAL under the New rule (which just says that F must be 1 or 2).
@thevanessakay
@thevanessakay Год назад
@@LSATLab Gotcha! Thanks.
@hornitorrincoperezoso2189
@hornitorrincoperezoso2189 Год назад
I have a question about when you test each substitution answer choice and choose (A) @15:00. I tested this answer the same way you did, but I accidentally assumed that it forced K before F in addition to making up for the replaced rule of F before J and L, and so I thought that it wouldn't have the same effect, but rather a stronger effect by forcing K before F. So this actually led me to a question: when doing the sufficient condition test, is it considered the same effect if the new rule has other effects in addition to creating the same effect as the old rule? Example: Here the effect we are replacing is F before J&L. What if one of the answers were to be sufficient to guarantee F before J&L AND something else like G before H? That wouldn't be considered the same effect, right? Even though the answer is a Must Be True, and a sufficient condition to infer the old rule, it would still not be the correct because the overall effect is different, correct? Basically, the sufficient condition test must show that you can infer the old rule and NOTHING ELSE?
@thatsseauxnish_
@thatsseauxnish_ 3 года назад
How does L have to be second?
@therealsherminator
@therealsherminator 3 года назад
I think you're asking about question 4, right? The question tells us that L is earlier than K. That means that L is earlier than K, J, H, and G. That forces F to go first and L to go second.
@thatsseauxnish_
@thatsseauxnish_ 3 года назад
Matt Sherman Amazing, thank you!
@SenseiLlama
@SenseiLlama 5 месяцев назад
The only thing left is repitition
Далее
Standard Ordering | LSAT Logic Games
32:35
Просмотров 29 тыс.
LSAT | Logic Games | General Approach
21:50
Просмотров 94 тыс.
when you have plan B 😂 @andreyreactions
00:11
Просмотров 4,4 млн
Stacked Ordering | LSAT Logic Games
37:09
Просмотров 28 тыс.
In/Out Grouping | LSAT Logic Games
27:33
Просмотров 25 тыс.
Struggling with Logical Reasoning? Watch this!
12:08
Просмотров 144 тыс.
How to Improve at LSAT: Dissecting Arguments
40:33
Просмотров 16 тыс.
Standard Grouping | LSAT Logic Games
28:32
Просмотров 27 тыс.
Paradox | LSAT Logical Reasoning
29:13
Просмотров 16 тыс.
How to Approach LSAT Reading Comprehension Questions
5:39
when you have plan B 😂 @andreyreactions
00:11
Просмотров 4,4 млн